Kettering Questions

¡Supera tus tareas y exámenes ahora con Quizwiz!

What should you do before attaching a flowmeter to an E-Cylinder?

"crack the tank"

Equation for duration of flow for E cylinder?

(0.3 X PSI) / L per min

Pulmonary vascular resistance formula

(MAP - PCWP) / CO

How to calculate ALVEOLAR minute ventilation :

(Vt - deadpace) X RR deadspace = weight in lbs

MIP for weaning =

- 20 or greater

Increase respiratory rate, increased depth, irregular rhythm :

- Kussmaul's breathing - Diabetes - Insulin / bicarb for metabolic acidosis

Ideal breathing pattern for improving ventilation (4) :

- slow, deep inspiration - Inspiratory pause - Exhalation is slow, and relaxed - Relax between breaths

Normal CoHb

0 - 1%

Atropine dose

0.5 Can be given up to 6X = 3 mg

Which of the following information may be obtained from a FVC maneuver during bedside pulmonary function testing? (select all that apply) 1. FEV1 2. PEFR 3. FRC 4. RV

1 and 2 only

Which of the following should the respiratory therapist consider when preparing for helicopter transport of a patient receiving mechanical ventilation? (select all that apply) 1. Select a ventilator that uses demand valves 2. Calculate oxygen cylinder duration of flow. 3. Selecting a ventilator that incorporates an internal air compressor.

1 and 2 only

A tympanic percussion note is usually present in which of the following conditions? (select all that apply) 1. gastric distension 2. pleural effusion 3. pneumothorax 4. endocarditis

1 and 3 Typanic = hyperesonant = air

A patient receiving pressure-controlled ventilation has acute hypoventilation with an ETCO2 of 70 torr. His vital signs include: heart rate 90/min, respiratory rate 18/min, SpO2 94%. Which of the following change(s) will address the situation? (select all that apply) 1. Increase the pressure limit 2. Increase the sensitivity 3. Increase the mandatory rate 4. Decrease the inspiratory time

1 and 3 only

A patient is on CPAP at 10 cmH2O and 0.30 FIO2 with the heated humidifier set at 40° C. As the gas is delivered to the patient through large bore tubing, which of the following will occur? (select all that apply) 1. Excess water will rain out 2. Humidity deficit will occur 3. Relative humidity will decrease 4. Relative humidity will remain 100%

1 and 4 only

At 1 minute post-delivery, a newborn has blue extremities with a pink body, heart rate is 90/min, respiratory rate is 20/min with a weak cry, cough reflex is present, and there is some flexion of the extremities. What APGAR score should be assigned?

1 minute APGAR = 6 Appearance - 1 Pulse - 1 Grimace - 2 (because of cough reflex) Activity - 1 Respiration - 1

A patient on VC ventilation has demonstrated auto-PEEP on ventilator graphics. Which of the following controls, when adjusted independently, would increase expiratory time? (select all that apply) 1. Tidal volume 2. Respiratory Rate 3. Inspiratory flow 4. Sensitivity

1, 2, and 3

The respiratory therapist notices that a mask CPAP system is unable to maintain the desired level of pressure. Which of the following might be causing the level to not be maintained? (select all that apply) 1. Sticking valve in the system 2. Loose fitting mask 3. Faulty humidifier connection 4. Leak around the airway cuff

1, 2, and 3 only

Treatment for ventilation issue (4)

1. BIPAP 2. PPV 3. Bag mask 4. Ventilator

Unable to establish an IV. When administering medication through ET tube; what additional things would we need to do (3) :

1. Double the dose 2. Flush with saline 3. Hyperventilate for 30 seconds

Activities of daily living (5)

1. Eating 2. Bathing 3. Dressing 4. Toilet use 5. Transferring

Treatment to increased ICP (4)

1. Hyperventilation (low Co2; 25 to 30) 2. Keep head of bed elevated 3. Increased O2 4. Mannitol - removes fluid

Path of pulmonary artery catheter (4)

1. Right Atrium (CVP) 2. Right Ventricle 3. Pulmonary Artery (PAP) 4. Pulmonary Capillaries (PCWP)

Things that are effected by oxygenation (5 vital signs)

1. heart rate 2. color 3. sensorium 4. PaO2 5. SpO2

Things that are effected by ventilation (5)

1. respiratory rate 2. tidal volume 3. chest movement 4. PaCO2 5. ETCO2

80/20 heliox factor =

1.8

Initial respiratory rate setting for starting a patient on vent :

10 - 20 bpm

High pressure is set ________ :

10 above the peak airway pressure

How often is IS / SMI performed by the patient?

10 breaths / hour

Sleep apnea can be defined as repeated episodes of complete cessation of airflow for

10 seconds or longer

Normal range for rate in an adult :

10 to 20 bpm

Low exhaled volume is set ______ :

100 below exhaled Vt

Desired FiO2 range for Emergency?

100%

A 55 year old patient admitted to the emergency department has a history of hypertension. The patient is conscious, diaphoretic and complaining of chest pain. The respiratory therapist's first response should be to

100% O2

A patient involved in a motor vehicle accident has sustained a long bone fracture and remains in traction. The patient suddenly complains of chest pain and develops tachypnea and tachycardia. The respiratory therapist should

100% O2

How much oxygen do we give burn victim, and near drowning?

100% O2

Treatment for CO poisoning (3)

100% O2 on non-rebreather HBO CPAP

Normal Hb :

12 - 16 (average = 15) 3 X 5 (rbc) = 15 (Hb)

ECG contains _____ leads and ____ electrodes

12 leads 10 electrodes (10 go on the patient)

Normal hemoglobin :

12 to 16

Minimum increase of ____ % and 200 mL in the FEV1 for testing bronchodilator treatment

12% and 200 mL

In preparing to perform a cardiopulmonary stress test on a 60-year-old man, the respiratory therapist must determine the target heart rate range for the patient. What is the patient's maximum heart rate? A. 130 beats/minute B. 160 beats/minute C. 190 beats/minute D. 220 beats/minute

160 bpm 220 - age of pt 220 - 60 160 bpm

Amioderone dose

1st dose = 300 2nd dose = 150

Adenosine dose

1st dose = 6 mg 2nd dose = 12 mg

What is the normal range for cardiac index in an adult?

2 - 4

PEEP range for infant being put on vent :

2 - 4 or same level that they are coming from

What is the normal range for cardiac index in an adult?

2 - 4 L/minute

NG tube placement

2 - 5 cm BELOW diaphragm

Initial PEEP setting for starting a patient on vent :

2 - 6 or same level that they are coming from

ET tube tip should be positioned :

2 - 6 inches above the carina

Normal L/S Ratio in infants

2 : 1 or higher

Where do we start HBO chamber?

2 ATA

A patient in the emergency department has frothy secretions, moist crackles, and tachypnea. The patient has marked dyspnea and a history of heart disease. Which of the following should the respiratory therapist recommend? 1. It Suction immediately 2. Administer 100% oxygen 3. Place in Fowlers position 4. Administer furosemide

2, 3, and 4 Flothy = sitting up (fowlers) Marked dyspnea = give 100% O2 Moist crackles = fluids = Lasix = furosemide Suctioning = does nothing to help

While performing diagnostic chest percussion, the respiratory therapist notes decreased resonance to percussion. Which of the following are potential causes of this finding? (select all that apply) 1. pneumothorax 2. pleural effusion 3. pneumonia 4. atelectasis

2, 3, and 4 only

Following insertion of a NASAL endotracheal tube, the respiratory therapist should inflate the cuff to ____ to ____

20 - 25

Normal cuff pressure range :

20 - 25 mmHg

Respiratory rate range for infant being put on vent :

20 - 30

Nitric Oxide (iNO) dosage range :

20 - 40

Normal Vd / Vt % range =

20 - 40%

What is the normal VD/VT ratio for a patient breathing room air?

20 - 40%

The respiratory therapist is preparing to administer inhaled nitric oxide to a neonate with respiratory distress. The most appropriate initial dose of iNO for this patient is : A. 5 ppm. B. 10 ppm. C. 15 ppm. D. 20 ppm.

20 ppm

Delivered FiO2 for non-rebreather?

21 - 100%

Oral intubation should be ____ to ____ mark at the patients lips.

21 to 25 cm

Which of the following values would be used to calibrate a galvanic fuel cell oxygen analyzer?

21% and 100%

PSI of a full E-cylinder :

2200

Desired FiO2 range for COPD patient?

24 - 28%

Before removing chest tube, how long do we clamp the tube for?

24 hours

Normal clotting time (APTT) :

24 to 32 seconds monitor heparin

Normal A-a Gradient values :

25 - 65

What is the total flow delivered to a patient when flow is set at 5 L / minute and the dilution control is set at 28%?

28% factor = 11 11 X 5 = 55 L / minute

Spirometry attempts : 2.92 3.08 2.96 How would you interpret this?

2920 3080 2960 They are all within 200 ml - Normal

Start I:E ratio at ______ with IRV

2:1

Formula for MAP

2X Diastolic + Systolic / 3

To drain AIR from the pleural space, the tube is placed :

2nd intercostal space / midclavicualr line

How many one-way valves does a non-rebreather have?

3

What range do we start CPAP =

3 - 5

A 68 kg (150 lb) patient has a spontaneous tidal volume of 450 mL and is breathing at a rate of 12 breaths/min. What is their minute alveolar ventilation?

3.6 L/min Vt - deadspace X rate 450 - 150 = 300 300 X 12 = 3600 ml = 3.6 L

Normal PETCO2

30

Desired FiO2 range for Therapeutic?

30 - 60%

What total flow is delivered to a patient if the air entrainment mask is set at 35% oxygen and the flowmeter is set at 6 L/min?

30 L / min FiO2 = 35% = flow factor = 5 5 X 6 = 30

Normal infant glucose

30 mg

How long is an SBT

30 minutes - 2 hours

HR equation based on ECG strip

300 / # of boxes between R waves

If FiO2 is 50% - what is the PAO2?

300 torr

Term infant is defined as born between :

38 and 42 weeks

Initial EPAP level range

4 - 6

Normal RBC count :

4 - 6 (average = 5)

Formula to estimate FiO2 for NC =

4% for every Liter/ minute

MEP for weaning =

40

Moderate Obstructive or Restrictive = __ to __ % of predicted

40 - 59%

Initial FiO2 setting for starting a patient on vent :

40 - 60% or same level that they are coming from

Normal Hct :

40 - 60% (average = 45) 3 X 15 (Hb) = 45 (Hct)

To produce capillary vasodilation, electrodes for transcutaneous monitoring must be heated to :

42 to 45 degrees

Transcutaneous monitoring temperature range

43 - 45 degrees

Normal PH2O =

47 torr

Normal ICP range

5 - 10 treatment after 20

Initial tidal volume setting for starting a patient on vent :

5 - 10 ml/kg IBW

Normal ICP =

5 to 10 mmHg

What value for the apnea-hypopnea index (AHI) is consistent with mild obstructive sleep apnea? A. Less than 5 B. 5 to 15 C. 16 to 30 D. Greater than 30

5 to 15

A-a gradient is 300 torr; what is the shunt %

5% + 5% + 5% + 5% answer = 20%

Normal WBC count =

5,000 to 10,000

To drain FLUID from the pleural space, the tube is placed :

5th - 7th intercostal space / midaxillary (armpit)

With a simple mask, the flow must be at least _____ L/minute to flush out CO2?

6 L/ minute

The following pulmanary function measurements have been determined for a 42-year-old male patient with asthma: VC: 5.1 L. FRC: 2.4 L. ERV: 1.4 L. VT: 0.5 L. IRV: 3.2 L. What should the therapist report as the total lung capacity (TLC)?

6.1 IRV + Vt + FRC 3.2 + 0.5 + 2.4 3.7 + 2.4 6.1

Mild Obstructive or Restrictive = __ to __ % of predicted

60 - 79%

Vd / Vt greater than _______ = vent time

60%

The correct intercostal location for needing insertion during thoracentesis?

7th to 8th ICS

Initial IPAP level range

8 - 12

Quality control must be performed for blood gas analyzers every _____ hours

8 hours

What is the normal range for the mean pulmonary artery pressure in an adult?

9 to 18

A 62-year-old female has the following arterial blood gas results on room air: pH: 7.37 PaCO2: 38 torr PaO2: 60 torr The co-oximeter is currently down for repair at this time. The physician has asked you to estimate the SaO2. You would estimate the SaO2 to be which of the following? A. 75% B. 80% C. 85% D. 90%

90% PaO2 in 60's = 90%

Normal SpO2

93 - 97%

Which of the following is a FALSE statement about self-inflating resuscitation devices? A. A reservoir is utilized to increase the delivered oxygen concentration. B. The therapist can sense changes in the patient's lung compliance and airway resistance. C. A compressed gas source is necessary for the device to operate. D. Excessive gas flow may cause the valve to malfunction.

A compressed gas source is necessary for the device to operate.

A patient is diagnosed with pneumonia and a large pleural effusion. Which of the following procedures should a respiratory therapist recommend?

A needle thoracentesis (used to quickly relieve the increased work of breathing caused by a pleural effusion and potentially aid in diagnosis)

What is a V /Q scan used for?

A normal ventilation scan with an ABNORMAL perfusion can indicates pulmonary embolism

Formula for A-a Gradient

A-aDO2 = PAO2 - PaO2

A P/F ratio less than 300 torr or greater - this means?

ALI

What controls ventilation on HFOV?

AMP or Hrzt (frequency)

Formula for alveolar minute volume?

AMV = Vt - DS X RR

This mode is just like IRV, but the patient can spontaneously breath.

APRV

A P/F ratio less than 200 torr - this means?

ARDS

Ground-glass appearance Honeycomb Reticulogranular

ARDS

When do we allow permissive hypercapnea

ARDS

Above which value for oxygenation index would indicate the need for ECMO in a newborn?

Above 40

A 28-week gestational age neonate is experiencing frequent apneic episodes with bradycardia. Which of the following should the respiratory therapist recommend? A. An FIO2 of 0.40 via oxyhood B. Suctioning the neonate during apnea episode C. Administering beclomethasone D. Administering theophylline

Administering theophylline Respiratory stimulant

Treatment for ARDS : Adults = Babies =

Adults : pressure control mode Babies : surfactant

Set of instructions documenting what treatment a patient would want if he/she was unable to make medical decisions :

Advanced Directives

If question states "end-stage" or "terminal" ; look at the chart for ___________ :

Advanced directives

Predicted spirometry values are based on 4 things :

Age Height Sex Race

Delivers precise FiO2; ideal for COPD patients? (2 names)

Air entrainment / Venturi mask

Continuous bubbling in chest drainage system =

Air leak

While monitoring a newborn utilizing a transcutaneous monitor, you notice a change in PtcO2 from 60 to 142 torr and simultaneously the (PtcCO2) changes from 37 to 2 torr. What is the most likely explanation for these changes?

Air leak

A patient on the general medical ward is on a 28% air entrainment mask with the flowmeter set at 5 L/min. What is the total flow delivered to the patient? A. 5 L/min B. 55 L/min C. 88 L/min D. 140 L/min

Air:Oxygen ratio for 28 = 10:1 Total flow = (5 x 10) + (5 x 1) = 50 + 5 = 55 L/m

Just the PIP went up, thats a _______ issue :

Airway

What do we sterilize bronchoscopes with?

Alkaline liquid solution

What do we sterilize nebulizers with?

Alkaline liquid solution

A male patient who is 175 cm (5 ft 9 in) and weighs 82 kg (180 lb) was receiving VC, A/C ventilation with a tidal volume of 600 mL. The patient's pH was normal. The patient was switched to PC, A/C ventilation at the same mandatory rate. Exhaled tidal volume is averaging 750 mL. A respiratory therapist should conclude the patient is most at risk for :

Alkalosis

Which of the following formulas calculate PAO2?

Alveolar air equation PAO2 = (PB - PH2O) FiO2 - PaCO2 / R

A spontaneously breathing 76-year-old patient who weighs 60 kg (132 lb) is on an FIO2 of 0.45 via a trach collar. He has had a change in his tidal volume from 600 mL to 300 mL and his respiratory rate went from 12 /min to 24 /min. Which of the following changed due to the change in the tidal volume and rate? A. Minute ventilation B. Alveolar ventilation C. Anatomical deadspace D. Mechanical deadspace

Alveolar ventilation Vt - deadspace = alveolar volume X RR

A spontaneously breathing 76-year-old patient who weighs 60 kg (132 lb) is on an FIO2 of 0.45 via a trach collar. He has had a change in his tidal volume from 600 mL to 300 mL and his respiratory rate went from 12 /min to 24 /min. Which of the following changed due to the change in the tidal volume and rate?

Alveolar ventilation change in rate and Vt MV = rate X Vt 600 X 12 = 7200 300 X 24 = 7200

Hb less than 12 =

Anemia

Treatment for pneumonia :

Antibiotics

At what value do we start intervention for increased ICP?

Anything over 20 mmHg

Indications for continuous mechanical vent (3)

Apnea Impending vent failure Acute respiratory failure

A post-operative thoracic surgery patient is having difficulty developing an effective cough. The respiratory therapist should recommend all of the following techniques to aid this patient in generating a more effective cough EXCEPT: A. coordinating coughing with pain medication. B. performing serial coughs. C. applying pressure to patient's abdomen during exhalation. D. "splinting" the incision area.

Applying pressure to patient's abdomen during exhalation

Provides continuous monitoring of systemic blood pressure and access for sampling :

Arterial line

Best index / indicator of oxygen transport to tissue?

Arterial oxygen content

When interviewing the patient, what kind of questions do we ask?

Ask open-ended questions Avoid Yes / No answers

Vent mode : the vent will deliver a minimum number of mandatory breaths. The patient can trigger breaths by decreasing pressure, flow, or volume :

Assist / Control (AC) Mode

Patient starts breath and ventilator finishes the breath

Assisted breath

Picture of ventilator graphics looks like a FISH =

Assisted breath

A V̇/Q̇ scan reveals ventilation and perfusion are equally decreased in the right mid-lung field. This finding is compatible with what disease?

Atelectasis

Patchy infiltrates Crowded vessels Crowded bronchograms Plate-like infiltrationsuffy infiltrate

Atelectasis

Tracheal deviation pulled to ABNORMAL (toward pathology) - name 3 causes :

Atelectasis Fibrosis Pneumonectomy

Flat or Dull percussive lungs sounds are associated with what 3 diseases?

Atelectasis Pleural Effusion Pneumonia

Tracheal deviation ; pulled towards abnormal side (towards pathology) (2) :

Atelectasis - give IS or IPPV Pulmonary Fibrosis

How to we evaluate IS treatment?

Auscultate breath sounds before and after

Ways to fix ventilation problem (4)

BIPAP PPV Bag mask Ventilator

Things that control perfusion (4)

BP Urine output Hemodynamics Temperature

Increased WBC (above 10,000) =

Bacterial infection

Test for low birth weight infants

Ballard score

If the PA catheter shows no difference in systolic and diastolic; what does that mean?

Balloon is inflated

Tip of the ET tube should be located (3)

Below vocal cords 2-6 cm above carina At level of aortic arch

Biological indicators used when cleaning equipment indicate ___________?

Biological indicators = sterilization has been met

Each breath has the same depth, but an increase in respiratory rate with periods of apnea

Biot's breathing

Most appropriate for surface disinfection

Bleach

What do we clean up blood with?

Bleach

Before each use Peak Flow Meter - make sure the arrow on :

Bottom of the numbered scale (zero)

How do we know bronchial hygiene is working?

Breath sounds

Oligopnea

Breathing that is shallower or slower, than normal

Treatment for medium crackles :

Bronchial hygiene - CPT - Percussion - Position

Treatment for medium crackles

Bronchial hygiene - CPT, percussion, and drainage

Medication used along side group counseling to help with mental support in a smoking cessation program :

Bupropion (Wellbutrin or Zyban)

A 65 kg (150 lb) patient requires a minute alveolar ventilation of 10 L/min. Which of the following set of parameters would be most appropriate? A. VT 600 mL, RR 16 /min, VD mech 0 mL B. VT 650 mL, RR 18 /min, VD mech 50 mL C. VT 700 mL, RR 20 /min, VD mech 50 mL D. VT 900 mL, RR 12 /min, VD mech 100 mL

C. VT 700 mL, RR 20 /min, VD mech 50 mL (Vt - deadspace) - anatomical DS X rate 700 - 150 = 550 - 50 = 500 500 X 20 = 1000 = 10 L

Enlarged heart on CXR :

CHF

Increased vascular markings means what disease?

CHF

Jugular vein distention = DX

CHF

What two diseases benefit most from bronchial hygiene?

CHF and bronchiectasis

C(a-v) 02 above 5% is a sign of decreased _____ :

CO

What type of patient would have an ABG that looks GOOD, but the patient looks BAD (3) ?

CO poisioning, Pulmonary embolism, and Anemia

CoHb with co-oximeter is used to assess?

CO poisoning

Above 20%

CO poisoning above ____ % = CO poisoning

Smoking cessation is monitored via :

COHb levels and Exhaled CO levels (FECO)

What type of patient would have an ABG that looks BAD, but the patient looks GOOD (1) ?

COP

Flat diaphragm on CXR :

COPD

What disease has high PCO2 with normal pH?

COPD

Indications for NPPV (3)

COPD, CHF, and Do not intubate

SBT utilizes what form of ventilation

CPAP

Which of the following therapies would be most appropriate for a spontaneously breathing patient who has atelectasis with a low SpO2 on an FIO2 of 1.0? A. Intermittent Positive Pressure Breathing (IPPB) B. Continuous Positive Airway Pressure (CPAP) C. Continuous Mechanical Ventilation (CMV) D. Intrapulmonary Percussive Ventilation (IPV)

CPAP

A 57-year-old male is admitted to the hospital complaining of dyspnea on exertion and a dry, non-productive cough. He reports his symptoms have been worsening for the past 2 months. The patient's respiratory rate is 28 with fine inspiratory crackles throughout the lung fields, but most prominent over the lower lung fields. Spirometry shows a restrictive defect. Which of the following should a respiratory therapist recommend? A. CT scan of the chest B. ventilation/perfusion study C. exercise stress testing D. inhalation challenge test

CT scan of the chest

Best exam for pulmonary embolism?

CT scan, then a pulmonary angiogram if inconclusive CT

A balloon-tipped, flow-directed catheter is positioned in the pulmonary artery with the balloon deflated. Which of the following pressures will be measured by the proximal lumen?

CVP

Normal Central venous pressure =

CVP = 2 - 6

Which of the following physiologic values would be present in a patient who has proper fluid balance? A. PCWP of 22 mm Hg B. CVP between 3 and 6 mm Hg C. urine output of 20 mL/hr D. increase in body weight

CVP between 3 and 6 mm Hg

A patient receiving continuous heated aerosol therapy with air by a T-piece develops subcutaneous emphysema around the tracheostomy site, neck, and chest. After noting unequal breath sounds, a respiratory therapist should recommend obtaining :

CXR

Normal arterial oxygen content?

CaO2 = 17 - 20

The following data are obtained for a patient: FVC = 5.79 L RV = 2.46 L FRC = 4.30 L ERV = 1.84 L IC = 3.95 L What is the patient's total lung capacity?

Calculated by adding FVC and RV 5.79 + 2.46 = 8.25

Pulse ox will read higher if ____________________ is present :

Carbon Monoxide Poisoning

Test to assess orthopnea?

Cardiac function test

A 59 year-old post CABG patient has a C(a-v)O2 that has increased from 5 mL/dL to 8 mL/dL. The respiratory therapist should report to the physician that the patient's : A. hemoglobin is increasing. B. cardiac output is decreasing. C. VD/VT ratio has increased. D. oxygen consumption has decreased.

Cardiac output is decreasing

Which form of cardiac emergency devices does the patient need to be sedated?

Cardioversion

Apnea due to ventilation issue :

Central sleep apnea NO CHEST MOVEMENT NO NASAL AIR FLOW

During IPPB therapy, a patient is meeting the inspiratory pressure in a shorter time frame than in the beginning of TX. This is most likely :

Change in airway resistance

A 80 kg (176 lb) male suffering from acute respiratory distress syndrome is being mechanically ventilated at the following settings: Mode: VC, A/C Tidal Volume: 500 mL Set Rate: 10 br/min Total Rate: 10 br/min PIP: 68 cm H2O FIO2: 0.45 PEEP: 8 cm H2O pH: 7.37 PaCO2: 44 torr PaO2: 89 torr HCO3-: 22 mEq/L The respiratory therapist should A. decrease PEEP. B. change to SIMV. C. add pressure support. D. change to pressure control.

Change to PC PIP = 68; higher than 50 and the patient has ARDS

A 65 year-old female patient with advanced emphysema comes to the ED and is placed on a nasal cannula at 6 L/min. On inspection, the respiratory therapist finds that the patient has become drowsy and less responsive since the oxygen therapy was initiated an hour ago. ABG on 6 L/min are: pH 7.33, PaCO2 64 torr, PaO2 85 torr, HCO3 35 mEq/L. Which of the following should the therapist recommend? A. Leave the patient on the cannula / continue to monitor. B. Change to a 24% Venti-mask and repeat ABG. C. Change to a simple oxygen mask and repeat ABG. D. Prepare the patient for endotracheal intubation.

Change to a 24% Venti-mask and repeat ABG

A patient receiving mechanical ventilation has a blood pressure of 60/40 mm Hg. The patient is currently on a continuous dopamine HCl infusion and the SpO2 monitor does not consistently display a waveform or saturation value. Which of the following is a respiratory therapist's best course of action? A. Perform serial blood gas analyses. B. Increase the dopamine infusion. C. Discontinue pulse oximetry. D. Change to a reflective forehead sensor.

Change to a reflective forehead sensor.

A 60 kg (132 lb) patient is being mechanically ventilated with the following settings: VC, A/C; VT 500 mL, respiratory rate 12/min, FIO2 1.00 and 10 cm H2O PEEP. The patient's peak airway pressure is 60 cm H2O and his SpO2 is 85%. A current chest x-ray shows diffuse bilateral infiltrates. Which of the following is the most appropriate action in order to reduce peak airway pressure? A. Increase the frequency. B. Change to airway pressure release ventilation. C. Decrease the inspiratory time. D. Increase PEEP to 15 cm H2O.

Change to airway pressure release ventilation

If you are using a HME and the secretion start to thicken?

Change to heated humidifier

You are trying to get a sputum sample via normal saline in a SVN, but its not working. What can you do next?

Change to hypertonic saline

Patient with a tracheostomy turns his head periodically and the Briggs adaptor (T-piece) pops off; what would you do?

Change to trach collar

Ambu bag fills rapidly, it collapses easily, and the chest is not moving. WHAT IS WRONG?

Check inlet valves (if the question asked what would you do - get another form of ventilation)

Tympanic percussion, increase pressures, asymmetrical chest movement - what can we do?

Chest tube with drainage

Associated with head injury or drugs; gradually increasing and decreasing respiratory rate with periods of apnea lasting up to 60 seconds

Cheyne-stokes breathing

Compensated ABG = _________ DX

Chronic

Mucoid - gray/white sputum =

Chronic Bronchitis

If a patient has clubbing of figures, what kind of patient is this?

Chronic COPD; avoid high levels of O2

Dynamic hyperinflation is a major concern when using Volume Control, Assist/Control ventilation in patients with which of the following conditions? A. Post-traumatic chest trauma B. Community-acquired pneumonia C. Spinal cord injury D. Chronic bronchitis

Chronic bronchitis

If there is continuous bubbling in the water-seal bottle of chest drainage system - what can we do?

Clamp the tube : If bubbling stops = check patient If bubbling continues = change the tube

When do we STOP Bronchial Hygiene (4) ?

Clear breath sounds / clear Xray Strong cough Ambulating well Hazards start to occur during tx

If aerosol device is not misting enough (4)

Clogged Low flow Low water Low temperature

How do we help prevent VAP (4)

Closed system suction CASS tube Use MDI instead of SVN Use heated wire curcuit

Form of suctioning indicated for patients with high oxygen or PEEP requirements :

Closed-system suction (in-line)

Used to diagnose carbon monoxide poisoning (2)

Co-Oximeter Hemoximeter

Function of the 1st bottle in chest drainage system =

Collection

When the pH is inside the normal range - is it compensated or non-compensated?

Compensated or Chronic

If both PIP and Plat increase, thats a ___________ issue :

Compliance

Bronchial breath sounds heard over the lung periphery indicate :

Consolidation

While a respiratory therapist auscultates a patient's chest, the patient repeats the words, "ninety-nine." Transmission of vocal sounds is increased in the right lower lobe compared to the other lung fields. Which of the following does this most likely indicate?

Consolidation

A post-operative patient is receiving mechanical ventilation in the ICU at the following settings: VC, A/C; VT 550 mL, respiratory rate 14/min, FIO2 0.50 and 10 cm H2O PEEP. Bedside monitoring results demonstrate that the PvO2 is 35 mm Hg and the SpO2 is 90%. The patient is alert and oriented with stable vital signs. Which of the following should the respiratory therapist recommend? A. Decrease the PEEP. B. Increase the FIO2. C. Initiation diuretic therapy. D. Continue to monitor closely

Continue to monitor closely

Excreted by kidneys and associated with kidney failure :

Creatinine

Steeple Sign Pencil Point Hourglass Sign

Croup

A patient with neuromuscular disease has been receiving ventilatory support for 4 months through a tracheostomy. The patient uses a speaking valve during the day, but receives VC, A/C ventilation at night. Which of the following should be used?

Cuffed trach tube

A 70 kg (154 lb) male is receiving non-invasive positive pressure ventilation (NIPPV) with a BiPAP ventilator set to 15 / 5. Arterial blood gases are as follows: pH: 7.33 PaCO2: 49 torr PaO2: 83 torr Which of the following IPAP / EPAP settings would best improve the patient's condition? A. 10 / 2 B. 15 / 10 C. 20 / 10 D. 20 / 5

D. 20 / 5 blow off CO2 and pressure support increases from 10 to 15

Coordination or timing concern not as important :

DPI

All of the following should increase during stress test EXCEPT :

Deadspace

During ventilator rounds in ICU, the respiratory therapist notes that the patient in Room 3 has AIR TRAPPING graphic displayed on her ventilator. What action should the therapist take?

Decrease inspiratory time

How to normalize a low CO2 (1)

Decrease respiratory rate

Ambu bag set to flush. You notice the valve is stuck in the open position. Your first response should be to :

Decrease the O2 flow

Heart failure causes ________ urine output

Decreased

Decreased compliance = ________ volume Increased compliance = ________ volume Increased airway resistance = ________ volumes Decreased airway resistance = ________ volumes

Decreased compliance = decreased volume Increased compliance = increased volume Increased airway resistance = decreased volumes Decreased airway resistance = increased volumes

The question is going to give you different times of the day. The values provided are going to show an increase in PIP, and the Plateau Pressure also rises - what is happening?

Decreasing lung compliance due to a disease INCREASE THE PEEP

A 53-year-old patient with a history of chronic bronchitis is expected to have difficulty clearing secretions during the post-op recovery days. The respiratory therapist should recommend which of the following to assist the patient with mobilization of their secretions? A. Deep breathing and coughing techniques B. IPPB with albuterol C. Aerosol therapy with racemic epinephrine D. MDI therapy with beclomethasone (Vanceril)

Deep breathing and coughing techniques

In a chest drainage system; what controls the amount of negative pressure in suction bottle?

Depth of tube in water

A patient receiving a nebulizer treatment with 3% saline complains of shortness of breath. A respiratory therapist should

Discontinue therapy

Which of the following should a respiratory therapist instruct a patient to use when cleaning the home CPAP mask and connecting tubing? A. acetic acid B. hydrogen peroxide C. dishwashing soap D. isopropyl alcohol

Dishwashing soap

The process of DESTROYING vegetative pathogens

Disinfection

Treatment for pleural effusion (3)

Diuretics Thorcentesis Chest tube

Treatment for pulmonary edema

Diuretics and O2

Breath sound normally heard over fluid filled lungs indicating pneumonia or pleural effusion :

Dull

Legal document that names a healthcare person responsible for making healthcare decisions for the patient :

Durable POA

A 60-year-old male patient weighs 80 kg (176 lb) and is on a volume-cycled ventilator at a set VT of 800 mL, a peak pressure reading of 65 cm H2O, exhaled VT measured at 760 mL, peak inspiratory flowrate is 60 L/min., a plateau pressure reading of 58 cm H2O and a PEEP of 5 cm H2O. Which of the following is most likely occurring at this time? A. Dynamic compliance is decreasing B. Static compliance is increasing C. Airway resistance is increasing D. Thoracic compliance is increasing

Dynamic compliance is decreasing

A 60-year-old male patient weighs 80 kg (176 lb) and is on a volume-cycled ventilator at a set VT of 800 mL, a peak pressure reading of 65 cm H2O, exhaled VT measured at 760 mL, peak inspiratory flowrate is 60 L/min., a plateau pressure reading of 58 cm H2O and a PEEP of 5 cm H2O. Which of the following is most likely occurring at this time? A. Dynamic compliance is decreasing B. Static compliance is increasing C. Airway resistance is increasing D. Thoracic compliance is increasing

Dynamic compliance is decreasing Increasing pressures = decreasing compliance

Good backup option for home oxygen deliver?

E or H Cylinder

Provides continuous visual image of the electrical activity of the heart :

ECG

Diagnostic tool for brain trauma / diagnostic

EEG

The patient is instructed to say "E" and it sounds like "A"

Egophony = pneumonia

The blender alarm is sounding. Which of the following is the most likely cause?

Either low on O2 or low on air

General malaise; think :

Electrolyte imbalance

What do you associate "general malaise" with?

Electrolyte imbalance

Wedge infiltrates on CXR = DX

Embolism

Following blunt chest trauma, a 35-year-old male is orally intubated and continuous mechanical ventilation is initiated. Physical assessment of the neck and chest reveal a midline trachea and significant reduction in thoracic expansion of the left chest. There are diminished breath sounds in the left lung compared to the right lung. These findings most likely indicate which of the following? A. flail chest on right thorax B. right tension pneumothorax C. endobronchial intubation D. subcutaneous emphysema

Endobronchial intubation

Thumb sign

Epiglottitis

What should we do if severe hemorrhage occurs after bronch. tissue biopsy? (2)

Epinephrine and compress the site with scope

What do we sterilize flowmeters with?

Ethylene Oxide (EOH)

What do we sterilize non-disposable HIV equipment with?

Ethylene Oxide (EOH)

What do we sterilize electronics with?

Ethylene Oxide (EOH) because its a gas

A 66-year-old patient with tuberculosis treated with antibiotics is no longer considered contagious and is ready for discharge from the hospital. During his hospitalization, he received IPPB therapy using a Bird Mark VII ventilator. What would be the most appropriate method of sterilizing the Bird Mark VII?

Ethylene oxide

Which method sterilizes by alkylation?

Ethylene oxide

O2 / Hb Dissociation Curve shifts to left - what does this do to the values?

Everything going down

Measurement of nitric oxide concentration in patients exhaled breath to assess steroid use for asthma :

Exhaled Nitric Oxide (FENO)

Test to monitor abstinence in cigarette smokers :

Exhaled carbon Monoxide (FECO)

Cloudy fluid / opaque

Exudate (infection)

In order to monitor compliance of hospital employees in a smoking cessation program, the respiratory therapist should monitor the employees' A. PaO2. B. FECO. C. PETCO2. D. FENO.

FECO

FENO levels go up = FENO levels go down =

FENO levels go up = getting worst FENO levels go down = getting better

Decreased _________ is the best indicator of obstructive lung disease :

FEV1

Which of the following values is used to evaluate an individual's response to inhaled bronchodilators?

FEV1

What volume remains in the lung at the end of a normal exhalation?

FRC

Decreased ______ is the best indicator of restrictive lung disease :

FVC

Best trial # for FVC - you add what 2 values?

FVC + FEV1

Allows patient to breathe through upper airway and speak when tube is plugged :

Fenestrated trach tube

Stable tachycardia TX 1st step 2nd step 3rd step

First = oxygen Second = Vaggle Third = Adenosine

Breath sound normally heard over areas of atelectasis :

Flat

Best position to prevent aspiration?

Flat lateral

Which of the following would be the best test to evaluate a patient's partial vocal cord paralysis? A. Flow volume loop B. MVV C. SB nitrogen elimination D. DCO

Flow Volume Loop

Flow factor for FiO2 of 28% = Flow factor for FiO2 of 40% = Flow factor for FiO2 of 60% =

Flow factor for FiO2 of 28% = 10 : 1 = 11 Flow factor for FiO2 of 40% = 3 : 1 = 4 Flow factor for FiO2 of 60% = 1 : 1 = 2

Spirometry : Flow is determined by ____ Volume is determined by ____

Flow is determined by FEV1 Volume is determined by FVC

Patient receiving aerosol treatment starts to breath deeper means =

Flow too low

Total Flow formula :

Flowmeter setting X Factor

Fine crackles indicates :

Fluid in alveoli - CHF or pulmonary edema

How to calculate shunting:

For every 100 torr of A-a gradient ; add 5% to a start of 5%

What is the volume of air remaining in the lungs after a normal expiration?

Functional residual capacity

Bubbling in the 3rd bottle is ________ :

GOOD no bubbling in 3rd bottle = turn up suction

How do we treat Non-compensated Respiratory alkalosis?

Give O2

Before putting a patient on HFOV - we need to?

Give sedation and NM blocker

Patient has an infection. Which of the following would QUICKLY determine the nature of that infection? A. Sputum culture B. Acid fast C. Gram Stain D. CXR

Gram Stain only takes an hour

Vital capacity for weaning =

Greater than 10 ml 2 X Vt

In the last 6 months, a patient with bronchiectasis who uses postural drainage at home has had three exacerbations requiring hospitalization. Which of the following should a respiratory therapist recommend? A. Insufflation/exsufflation device B. Nebulized ipratropium bromide (Atrovent) C. Inhaled corticosteroid D. HFCWO

HFCWO This will loosen secreations

Which of the following should a respiratory therapist emphasize as a critical infection control step to a patient who will administer aerosol therapy at home?

Hand washing before and after therapy

If the RBC count goes down; what else goes down?

Hb and Hct

A 52 year-old post-operative patient's chest radiograph demonstrates infiltrates in the posterior basal segments of the lower lobes. Which of the following is the appropriate postural drainage position? A. Head down, patient supine with a pillow under knees B. Patient prone with a pillow under head, bed flat C. Patient supine with a pillow under knees, bed flat D. Head down, patient prone with a pillow under hips

Head down, patient prone with a pillow under hips

Elevated S-T segment indicates

Heart Injury

Which of the following devices can provide 100% humidity at body temperature?

Heated wick humidifier

A 47-year-old patient admitted for sepsis has a CaO2 value of 12.5 vol%. The patient does not appear cyanotic. Which of the following would be the most important to further evaluate the patient's oxygenation status? A. PaO2 B. SaO2 C. Hb D. PAO2

Hemaglobin CaO2 value of 12.5 vol% = anemia

A 36 year-old fireman was trapped and subsequently rescued from the collapse of a burning building. Which of the following devices would be appropriate to accurately assess his oxygenation status? A. capnograph B. pulse oximeter C. blood gas analyzer D. hemoximeter

Hemoximeter

Treatment for pulmonary embolism :

Heparin

End-stage renal Dx - patient wants to die a home - where do we refer to?

Hospice

Coughing with an OPEN glottis :

Huff cough

Form of coughing that is more effective in patients with COPD or head trauma

Huff cough

Hyperresonant or Typanic percussive lungs sounds are associated with what 3 diseases?

Hyperinflation (too much air) Pneumothorax Emphysema

Breath sound normally heard in lungs with pneumothorax or emphysema :

Hyperresonant

If waves on capnogaphy are very frequent; what is happening (2)

Hyperventilation (PETCO2 = 20) Increased dead space

Hazard of IPPB =

Hyperventilation causes lightheaded and tingling

What type of breathing can we have the patient do for increase ICP?

Hyperventilation to decrease CO2

All values of PA catheter UP =

Hypervolemia

A patient in the intensive care unit has the following hemodynamic measurements: CVP (mm Hg): 12 PAP (mm Hg): 48/25 PCWP (mm Hg): 18 MAP (mm Hg): 104 Cardiac output (L/min): 7.1 Cardiac index (L/min/m2): 3.7 These results are consistent with: A. Hypervolemia B. Congestive heart failure C. Pulmonary hypertension D. Cor pulmonale

Hypervolemia Multiple values are increased

A patient who has significant decreases in airflow during sleep but does not have a complete cessation of breathing is having what type of episodes? A. Obstructive sleep apnea B. Dyspnea C. Hypopnea D. Central sleep apnea

Hypopnea

When would we NOT use IPPB? (3)

Hypotension, untreated pneumothorax (without chest tube) and increase ICP

All values of PA catheter DOWN =

Hypovolemia

A patient in the ICU receiving mechanical ventilation has just undergone a fiberoptic bronchoscopy in which a tissue biopsy was collected. Immediately after the procedure, the respiratory therapist notes that the peak inspiratory pressure on the ventilator has increased. Potential causes for this include all of the following EXCEPT : A. hypoxemia. B. pneumothorax. C. pulmonary hemorrhage. D. bronchospasm/laryngospasm.

Hypoxemia

The most serious complication associated with airway suctioning is :

Hypoxemia

Most severe hazard of suctioning :

Hypoxemia (question will say : HR increases, tachycardia, PVC's)

PaO2 below 80 Disease? Treatment?

Hypoxemia Increase FiO2 up to 60%

Inhale or exhale with IS?

INHALE ONLY!

Pressure support equation for CPAP

IPAP - EPAP

Bird Mark 7 ventilator is used for?

IPPB

Decreased WOB to decrease accessory muscle use in patients with COPD :

IPPB

Device used to promote a cough

IPPB

Used to prevent or treat pulmonary edema

IPPB

A 65 kg spinal cord injured patient has developed atelectasis. His inspiratory capacity is 30% of his predicted value. What bronchial hygiene therapy would be most appropriate initially?

IPPB with normal saline

How do we calculate VC :

IRV + ERV + Vt

Quality control test on ABG machine shows all values are within 2 standard deviations : A. Trend B. Normal C. In Control D. Out of Control

In control

Where do we place a HME?

In the ventilator circuit between the Y and the patient

Prevention or treatment of atelectasis for patients who are willing and able to spontaneously take a breath :

Incentive Spirometry (test will call it Sustained Maximal Inspiration - SMI)

The best method of treating contaminated DISPOSABLE supplies :

Incineration

PaO2 below 80 - but patient is already on 60% Treatment?

Increase CPAP or PEEP

How to increase a low PO2 - FIRST

Increase FiO2 up to 60% (increments of 5-10)

A fixed-wing medical transport with an unpressurized cabin has ascended to 10,000 ft while transporting a patient with COPD. The patient is receiving nasal oxygen at 2 L/min and becomes agitated and confused. A respiratory therapist should :

Increase O2 flow

How to increase a low PO2 - SECOND

Increase PEEP (increments of 2-5) increase until patient gets better or worst

The question is going to give you different times of the day. The values provided are going to show an increase in PIP, but the Plateau Pressure remains the same - what is happening?

Increase airway resistance due to secretions or bronchospasm PIP - Plat (look at first and last time)

An increase in the VD / VT ratio indicates 2 things :

Increase in deadspace Pulmonary embolism

Patient on vent. and high pressure alarm is sounding with each inspiration. Wide fluctuations in pressure - what can we do?

Increase peak flow rate

Increase pressure = Dynamic compliance and Static compliance is ___________________________

Increase pressure = compliance is decreasing

How to normalize a high CO2 (3)

Increase respiratory rate Increase Vt Remove deadspace

Picture of ventilator graphics is lying down :

Increase the PEEP to 10 ARDS / PE

An infant on a high frequency jet ventilator (rate of 150 b/m) has the following arterial blood gases: pH: 7.30 PaO2: 60 torr PaCO2: 50 torr HCO3-: 21 mEq/L Which of the following changes would best help to improve these results? A. Increase the inspiratory time B. Increase the drive pressure C. Increase the frequency to 190 br/min D. Increase the FIO2

Increase the drive pressure We want to lower the CO2 by increasing the pressure. Driving pressure is the same of amp

Increase vascular markings = DX Decrease vascular markings = DX

Increase vascular markings = CHF Decrease vascular markings = pneumothorax

Which of the following findings is LEAST compatible with hyperlucency as seen on a chest x-ray? A. Increased fremitus B. Decreased intensity of breath sounds C. Diminished diaphragmatic excursion D. Hyperresonance to percussion

Increased fremitus

How does HBO help patients

Increases barometric pressure

Significant Q waves indicate

Infarction

A patient with COPD and pneumonia is receiving VC, SIMV. The following values are observed: FIO2 = 0.28 Mandatory rate = 8 Total rate = 16 VT = 700 mL Spontaneous VT = 425 mL Pressure support = 5 cm H2O PEEP = 4 cm H2O Vital signs have been stable for 48 hours and the following blood gas values are available: pH 7.33 PaCO2 54 torr PaO2 78 torr HCO3- 28 mEq/L BE +1 mEq/L Which of the following should a respiratory therapist recommend? A. Change to PC ventilation. B. Titrate PS to achieve VT of 700 mL. C. Extubate to an FIO2 of 0.40 with an aerosol mask. D. Initiate a spontaneous breathing trial.

Initiate a spontaneous breathing trial. (patient has been stable for 24 hours; COPD pt so we allow those values)

How do we treat Non-compensated Respiratory acidosis?

Initiate ventilation

______ is indicated by an elevated S-T segment :

Injury

Bleeding or blood in chest means we need to do what?

Insert Chest tube

Greater than 20% peumothorax =

Insert Chest tube

How do we calibrate a blood gas machine?

Inserting solutions with known values

What volume does the IS / SMI record?

Inspiratory capacity

Vent mode : patients requiring FiO2 higher than 60%, PIP higher than 50 and PEEP higher than 15

Inverse Ratio Ventilation

Gamma rays are used to sterilize pre-packed equipment :

Irradiation

Depressed or inverted T wave indicates

Ischemia

______ is indicated by a depressed or inverted T wave :

Ischemia

Forced Exp. Flow 200-1200 represents what part of the airways?

Large airway

Coarse crackles indicates :

Large airway secretions

No cuff Permanent airway Shorter than trach.

Lary. tube

Surgical removal of the patients larynx :

Laryngectomy

What is the most appropriate position for a female patient who is 5'3" tall, weighs 200 kg and is complaining of difficulty breathing?

Lateral Fowlers

Diagnostic tool to ID airway obstruction in children with croup or epiglottitis :

Lateral Neck Xray

The respiratory therapist is calibrating a spirometer and checking the volume with a 3.0 liter super syringe. The volumes recorded are: 2.85 L, 2.8 L, and 2.8 L. Based upon the information obtained - what is wrong?

Leak

What hand holds the laryngoscope?

Left hand

If all values are normal except PCWP; this indicates :

Left heart issue

Severe Obstructive or Restrictive = __ to __ % of predicted

Less than 40%

MOBILE oxygen delivery option

Liquid Oxygen system

Describes treatment patient would want if he/she became terminally ill; does NOT appoint someone else to make medical decisions :

Living Will

Transport cylinder is showing pressure of 1,000 and flow of 4 L/min - what should we do?

Look for obstruction

Before bringing any equipment into a patients home; what do we look for?

Look for properly grounded 3-PRONG electrical outlets

A bubble humidifier is a ______ device :

Low Flow

An air/oxygen proportioner is used to provide an FIO2 of 0.55 by a non-rebreather mask. The blender alarm is sounding. Which of the following is the most likely cause? A. Low oxygen inlet pressure B. High air inlet pressure C. Faulty humidifier bottle connection D. Excessive flow to the non-rebreather mask

Low oxygen inlet pressure alarm will sound if pressure is low

The peak inspiratory pressure on a pressure limited, time-cycled ventilator was changed from 20 cmH2O pressure to 25 cmH2O pressure. Which of the following alarms need to be adjusted at this time?

Low pressure alarm need to know if there is a leak

Normally the PETCO2 will read ________ than the arterial PCO2

Lower

Picture of ventilator graphics looks like a BIRDS BEAK =

Lower the Vt Over distention

Treatment for Atelectasis (4)

Lung expansion therapy : IPPB CPAP PEEP SMI

A 68 kg patient is receiving mechanical ventilation at Vt = 600 ml and rate = 10. What is the minute ventilation?

ME = Vt X RR ME = 600 X 10 ME = 6000 ml

PFT device used to assess patients ability to clear secretions (coughing) :

MEP

______ is diagnosed by significant Q waves

MI

While reviewing a patient's medical record the respiratory therapist notes that the patient has a Hb level of 14g/dL and an SpO2 of 94%. The patient is receiving oxygen by simple mask at 6 L/min. The respiratory therapist should recommend: A. Non-rebreather mask at 10 LPM B. A unit of whole blood C. CPAP therapy D. Maintain current therapy

Maintain current therapy Hb and SpO2 are normal

If there is a sudden fall in readings of SpO2 without a change in patients condition

Malfunctioning probe

Breath that ventilator controls all variables; patient does nothing :

Mandatory breath

Picture of ventilator graphics looks like a FOOTBALL =

Mandatory breath

If mechanical percussor malfunctions - what do you do next?

Manual percussion

A patient has been diagnosed with vocal cord polyps. Which pulmonary function test would best evaluate degree of impairment?

Maximum expiratory flow-volume curve (MEFV)

A patient in the intensive care unit has the following hemodynamic measurements: CVP (mm Hg): 12 PAP (mm Hg): 49/25 PCWP (mm Hg): 18 MAP (mm Hg): 99 Cardiac output (L/min): 7.1 Cardiac index (L/min/m2): 3.7 What is the mean pulmonary artery pressure?

Mean PAP = (2 X diastolic) + systolic / 3 MPAP = 2 X 25 50 + 49 = 99 99 / 3 = 33

What controls oxygenation on HFOV?

Mean airway pressure (PAW)

Medium crackles indicates :

Middle airway secretions

The most common complication associated with fiberoptic bronchoscopy via the nasal route is

Mild bleeding

Minimum FEV1 / FVC

Minimum 70% exhaled in 1 second

Apnea Index (AHI_) Moderate =

Moderate = 16 - 30 so less than 16 is mild and more than 30 is severe

Diffused infiltrates - treatment?

More PEEP for atelectasis

Troponin levels higher than 0.1 means

Myocardial Infarction

A 60 year-old male is admitted to the ED with chest pain. The CBC and electrolytes are normal. Troponin level is 0.4 ng/mL. The physician should report to the patient that he is suffering from : A. pulmonary embolism. B. gastroesophageal reflux. C. myocardial infarction. D. valvular stenosis.

Myocardial infarction

Delivered FiO2 for nasal cannula?

NC = 24-45%

Flow for nasal cannula?

NC flow = 1 - 6 L/minute

A 5-year-old patient requires low-flow oxygen therapy. Which of the following devices should the respiratory therapist recommend?

Nasal Cannula 5 years old = comfortable for them

Type of diseases that use negative pressure ventilation

Neurmuscular Dx Amy. lateral sclerosis (ALS) Guillain Barre Myasthenia Gravis

Potent vasodilator to improve pulmonary blood flow and PaO2 :

Nitric Oxide

Treatment for Cor Pulmonale

Nitric Oxide

When the pH is outside the normal range - is it compensated or non-compensated?

Non-compensated or Acute

Heliox is delivered via what device?

Non-rebreather

High flow device used to deliver 100% O2 to patients with pneumothorax, CO poisoning, CHF, or burn?

Non-rebreather

The respiratory therapist receives an order to administer a mixture of 80% helium/20% oxygen to a 40 year-old patient in an acute asthmatic episode. Which of the following devices would most effectively deliver the gas mixture to the patient?

Non-rebreather

A trauma patient in the ED is spontaneously breathing oxygen via nasal cannula at 2 L/min. Vital signs are heart rate 110/min, respiratory rate 32/min, blood pressure 90/60 mmHg. The pulse oximeter is reading 88%. Which of the following should the respiratory therapist recommend to maximize the patient's FIO2?

Non-rebreathing mask

Eupnea - breathing pattern :

Normal breathing

Continuous bubbling in the SUCTION bottle =

Normal for chest tube

If the tops of the R waves are 3-5 boxes =

Normal rate (60-100)

Eupnea

Normal respiratory rate, depth and rhythm

When you see SOB in question; think to give _________

O2

Treatment for croup (3)

O2 Cool aerosol Race Epi

Treatment for fine crackles :

O2 and Diuretics - CHF - Pulmonary edema

Treatment for fine crackles (3 things)

O2, positive pressure therapy, and diuretics

High pressure alarm - consider 2 things

Obstruction Increase airway resistance DX

"Shark fins" on capnography (non-flat tops)

Obstructive (COPD)

CBABE :

Obstructive : CF Bronchitis Asthma Bronchietasis Emphysema

Prevent cross-contamination when using a MIP on multiple patients :

One-Way valve

A 4-year-old child has been brought to the emergency room with an acute infection, high fever, marked stridor and drooling. Which of the following should the therapist recommend at this time? A. Arterial blood gas B. Oral intubation C. Lateral neck X-ray D. 40% oxygen via cool mist tent

Oral Intubation If you can tell is epiglottitis = intubate

Quality control test on ABG machine shows 2 or more plots moving outside the 2 SD :

Out of control Check equipment / maintenance

A 13 year-old patient in the ED is complaining of dyspnea, chest tightness, and a loose productive cough. The patient has a respiratory rate of 28 breaths/minute and bilateral wheezing in the lungs. What treatment should the respiratory therapist initiate? A. levalbuterol B. oxygen C. salmeterol D. PEP

Oxygen

Treatment for PVC :

Oxygen

Treatment for sinus tachycardia with a pulse :

Oxygen

A patient in the intensive care unit has the following hemodynamic measurements: CVP: 4 mm Hg PAP: 48/16 mm Hg PCWP: 8 mm Hg MAP: 92 mm Hg Cardiac Output: 5 L/min. Cardiac Index: 2.5 L/min/m2 The respiratory therapist should recommend the administration of :

Oxygen PAP = high = lung dx = oxygen

Moderate stridor with dyspnea treatment :

Oxygen first, then race Epi.

EPAP controls

Oxygenation

The most COMMON life function problem?

Oxygenation (if on the exam they give you a patient and they are ventilation - breathing- but you cant figure out whats wrong; give them oxygen)

When do we increase IPAP and EPAP?

Oxygenation issue

Which of the following will determine the partial pressure of oxygen in the alveolus? A. (Hb x 1.34 x SaO2) + (PaO2 x .003) B. (PaCO2 - PECO2) / PaCO2 C. (PAO2 - PaO2) .003 / [(PAO2 - PaO2) .003] + C(a-v)O2 D. (PB - PH2O) FIO2 - (PaCO2 / 0.8)

PB - PH2O) FIO2 - (PaCO2 / 0.8)

If the pre-ductal Pao2 is 15 torr higher than the post-ductal PaO2; then the patient has :

PDA

Right arm (pre-ductal) ABG shows higher PaCO2, than post-ductal site :

PDA

A 50 kg (110 lb) patient is being mechanically ventilated with the following settings: VC, A/C, VT 400 mL, respiratory rate 14/min, FIO2 0.60 and 10 cm H2O PEEP. The chest radiograph demonstrates diffuse bilateral radiopacity. ABG results are: pH 7.36, PaCO2 45 torr, PaO2 50 torr, and HCO3- 28 mEq/L. The respiratory therapist should increase the : A. PEEP. B. FIO2. C. expiratory time. D. respiratory rate.

PEEP

Patient was in MVA and has a ICP monitor showing ICP of 18 - You CANNOT increase ___________ :

PEEP Increase FiO2 instead of PEEP even if its at 60%

When reducing settings on mechanical vent for weaning; work backwards - decrease _______ first

PEEP first

A 28-week gestational age infant with severe respiratory distress syndrome is being mechanically ventilated in the PC, SIMV mode at the following settings: Flow rate: 6 L/min Set rate: 32 br/min. PIP: 28 cmH2O FIO2: 0.70 PEEP: 5 cm H2O I time: 0.8 seconds Umbilical arterial blood gas results reveal: pH: 7.40 PCO2: 39 torr PO2: 42 torr HCO3-: 23 mEq/L Based on the above information, the respiratory therapist should change the :

PEEP to 7 cm H2O Cannot go over 60 for FiO2 Once you get to 60 ; start raising PEEP

A 3-year-old child with cystic fibrosis is being mechanically ventilated at the following settings: PIP: 34 cm H2O I time: 1.0 second Rate: 22 /min. FIO2: 0.60 PEEP: 6 cm H2O Mode: PC, SIMV Arterial blood gas results show: pH: 7.36 PaCO2: 44 torr PaO2: 49 torr HCO3-: 24 mEq/L The respiratory therapist should adjust the A. PEEP to 8 cm H2O. B. FIO2 to 0.65. C. set rate to 24 /min. D. PIP to 36 cm H2O.

PEEP to 8 FIO2 is already at 60%. If there is still an oxygenation issue, we raise the PEEP after 60% FIO2.

What is the cheapest form of airway clearance therapy?

PEP therapy

Pressure support equation for VC-SIMV

PIP - Plat

Cannot trust the _______ on a heel stick :

PO2

The respiratory therapist obtains a SpO2 reading of 90% on a patient receiving oxygen therapy via 50% venti-mask. This would indicate a PO2 value of approximately?

PO2 = SpO2 - 30 PO2 = 90 - 30 PO2 = 60 torr

Duration of flow formula :

PSI X tank factor / liter flow (Full tank = 2200 PSI)

When do we REMOVE deadspace?

PaCO2 above 45

When do we ADD deadspace?

PaCO2 below 35

SaO2 - 30 =

PaO2

Pack Years equation

Pack years = # of packs /day X # of years smokes 40 packs/day X 10 years = 40 pack years

Low pH, high CO2, and high HCO3 - what is happening?

Partially compensates Respiratory Acidosis

FECO level below 7 =

Patient is a Non-smoker

FECO level of 7 or above =

Patient is a smoker

pH = 7.3 CO2 = 80 PO2 = 80 HCO3 = 38 What is wrong and how do we treat?

Patient is not ventilating, but they are oxygenating. The HCO3 is very high Over oxygenated COPD patient Reduce FiO2

Which of the following patients would most likely benefit from pressure support ventilation? A. An intubated patient with an absent respiratory drive. B. A patient on SIMV with set rate of 12; total rate of 24 C. A patient with acute lung injury. D. A patient who requires short-term post-operative ventilatory support.

Patient on SIMV with set rate of 12; total rate of 24

Which of the following is the best monitor of bronchodilator therapy effectiveness for a patient with asthma?

Peak Flow Measurements

Asthma patient exhales forcefully through the PFT device which has a movable indicator :

Peak Flow Meter

When using this device, blow out as hard and quickly as possible until lungs are nearly empty :

Peak Flow Meter (repeat 3X - all numbers must be close)

Peak Flow Meter Green Zone = Peak Flow Meter Yellow Zone = Peak Flow Meter Green Zone =

Peak Flow Meter Green Zone = 80 - 100 (normal) Peak Flow Meter Yellow Zone = 50 - 79 (caution) Peak Flow Meter Green Zone = less than 50 (medical alert - airway narrowing)

The patient in ICU Bed 6 is noted to have a meniscus in the left chest with a blunted left costophrenic angle on the morning chest radiograph. On physical exam, the respiratory therapist finds that the breath sounds are decreased on the left with a dull percussion note. What treatment should the therapist recommend? A. Insertion of an anterior chest tube. B. Bronchoalveolar lavage. C. Needle aspiration of the 4th left intercostal space. D. Perform a left posterior thoracentesis.

Perform a left posterior thoracentesis

An obese patient is being weaned from the ventilator following cholecystectomy. Arterial blood gas results reveal: pH: 7.40 PaO2: 96 torr PaCO2: 43 torr SaO2: 94% The patient is currently on: Tidal volume 650 mL, SIMV mandatory rate 4 /min, PEEP 5 cm H2O, and FIO2 0.35 even though she is not fully conscious. What should the respiratory therapist recommend at this time? A. Extubate B. Re-evaluate in 1 hour C. Place on a T-piece at a FIO2 0.45 D. Repeat arterial blood gases in 30 minutes

Place on a T-piece at a FIO2 0.45 The vent is no longer needed, but we want to wait for her to wake up a little bit.

An obese patient is being weaned from the ventilator following cholecystectomy. Arterial blood gas results reveal: pH: 7.40 PaO2: 96 torr PaCO2: 43 torr SaO2: 94% The patient is currently on: Tidal volume 650 mL, SIMV mandatory rate 4 /min, PEEP 5 cm H2O, and FIO2 0.35 even though she is not fully conscious. What should the respiratory therapist recommend at this time? A. Extubate B. Re-evaluate in 1 hour C. Place on a T-piece at a FIO2 0.45 D. Repeat arterial blood gases in 30 minutes

Place on a T-piece at a FIO2 0.45 ventilator is no longer needed

What type of pressure shows that Static compliance is increasing?

Plateau pressure is decreasing

Most common reason we use thoracentesis?

Pleural effusion

(Blunted) Obliterated cosophrenic angel; what does this mean, and how do we treat?

Pleural effusion - Thorocentesis - Diuretics - Chest tube

Delivers 100% O2 gas source in an emergency situation :

Pneumatically powered resuscitation Device

Air bronchograms on CXR :

Pneumonia

The most probable cause of air bronchograms and increased tactile frenitus on a chest x-ray is :

Pneumonia

While assessing a patient's breath sounds the respiratory therapist notes that when the patient says "ninety-nine", it sounds very loud through the stethescope. This would be associated with which of the following conditions?

Pneumonia

Consolidation on CXR (2) :

Pneumonia / Pleural effusion

The most common hazard associated with insertion of a central venous catheter is

Pneumothorax

Position for ARDS = Position for CHF = Unilateral disease =

Position for ARDS = prone Position for CHF = fowlers Unilateral disease = good lung down

Confirms that collateral blood flow is present :

Positive Modified Allens test

Low pressure alarm - consider 3 things

Pt disconnect Leak Low flow

A spiral CT scan with contrast may be used to diagnose :

Pulmonary Embolus

PVR over 2.5 mm = __________

Pulmonary HTN / Lung Dx

A mixed venous blood sample is needed to determine the oxygen consumption of the tissues. The mixed venous blood sample should be obtained from the :

Pulmonary artery

Fluffy infiltrates Butterfly pattern Bat-wing pattern

Pulmonary edema

Decrease tactile frenitus = DX

Pulmonary effusion

A patient in the intensive care unit has the following hemodynamic measurements: CVP: 9 mm Hg Mean PAP: 24 mm Hg PCWP: 9 mm Hg MAP: 93 mm Hg Cardiac output (QT): 5.4 L/min Cardiac index (QI): 2.9 L/min/m2 These results are consistent with A. right heart failure. B. left heart failure. C. pulmonary embolism. D. normal cardiac function.

Pulmonary embolism mean PAP = 24; usually 13 - PAP leads to lungs

Senses the start of inspiration and delivers oxygen only during inspiration :

Pulse Dose Oxygen Delivery

Used in the place of a flowmeter with low flow oxygen devices and connected to 50 PSI gas source :

Pulse Dose Oxygen Delivery

Furosemide (Lasix) and oxygen therapy are initiated for a patient with pulmonary edema. Which of the following should a respiratory therapist recommend? A. monitoring of electrolyte levels and a PA catheter B. a CBC and pulse oximetry C. pulse oximetry and monitoring of electrolyte levels D. a PA catheter and a CBC

Pulse oximetry and monitoring of electrolyte levels

RQ (metabolic studies) RQ for Carbs = RQ for Fats = RQ for Proteins =

RQ for Carbs = 9's to 1 RQ for Fats = 7's RQ for Proteins = 8's

Calculate RSBI

RR / Vt Vt needs to be in liters

When do we stop an SBT (6)

RR increases HR increases by 20 beats Cardiac arrhythmia Anxiety O2 desaturation Change in BP

Quality control test on ABG machine shows 1 point that falls outside the 2 SD range :

Random Error DO NOTHING

An alert adult patient is receiving CPAP with PS of 5 cm H2O and an FIO2 of 0.28. The patient has a heart rate of 88 and a respiratory rate of 25. Blood gas analysis results are as follows: pH = 7.43 PaCO2 = 35 torr PaO2 = 95 torr HCO3- = 23 mEq/L BE = -1 mEq/L Which of the following is a respiratory therapist's most appropriate action? A. Recommend extubation of the patient. B. Assess pulmonary mechanics. C. Wean if the shunt is less than 5%. D. Maintain current ventilator settings.

Recommend extubation of the patient. (Patient is ready for extubation. Assessing pulmonary mechanics will provide additional clinical information; however, it is unnecessary in this patient)

While administering 3.5 mg of albuterol to a patient with asthma in the ICU, the respiratory therapist notes that the patient's heart rate increases from 120 to 150 beats/minute. What is the appropriate modification for the next treatment for this patient? A. Reduce the dose of albuterol. B. Discontinue the treatment. C. Change to 3 puffs of beclomethasone dipropionate. D. Change to 0.63 mg of levalbuterol (Xopenex®).

Reduce the dose of albuterol

Trach. tube is obstructed; unable to pass even the smaller catheter :

Remove and insert new tube

A 44 year-old patient who suffered a cerebral vascular accident has been moved from Neuro-ICU to the step-down unit. He becomes diaphoretic and his SpO2 suddenly drops from 95% to 88% on a 32% tracheostomy collar. His heart rate is 115/min, respiratory rate is 42/min and his breath sounds are very diminished. The respiratory therapist is unsuccessful in attempting to pass a 12 Fr suction catheter. The therapist should : A. increase the suction pressure to 120 mm Hg. B. change to a 10 Fr suction catheter. C. replace the tracheostomy tube. D. orally intubate the patient.

Replace the tracheostomy tube

If patient has a laryngectomy tube and needs to be put on a vent :

Replace with cuff ET tube if vent is required

What is the volume of air remaining in the lungs after the expiratory reserve volume is exhaled?

Residual volume

Normal air-filled lung; this gives a hollow sound :

Resonant

Noninvasive Positive Pressure Ventilation (NPPV) is contraindicated in the management of which of the following conditions? A. Acute exacerbation of COPD B. Cardiogenic pulmonary edema C. Respiratory / cardiac arrest D. Premature extubation

Respiratory / cardiac arrest

Guidelines for administering respiratory care procedures to specific patients; when to start and stop treatments :

Respiratory care protocols / Therapist driven protocols

PFT device that measures FLOW (2)

Respirometer Pneumotachometer

If patient is weaning from Nitric Oxide and starts to have adverse effect; what would you do?

Return to previous iNO level

Unilateral wheezing indicates foreign body obstruction - what can we do?

Ridged Bronchoscopy

What is considered the pre-ductal site on infants?

Right arm

Increase CVP can cause (3)

Right heart failure Cor pulmonale Tricuspid valve stenosis

Where will you see the PA catheter on the CXR?

Right lower lung field

Flow calibration of PFT devices are done with :

Rotameter

Accuracy of flowmeter can be verified via? (2 ways)

Rotameter or calibration

A patient in the intensive care unit has the following hemodynamic measurements: CVP: 6 mm Hg Mean PAP: 13 mm Hg PCWP: 7 mm Hg MAP: 86 mm Hg Cardiac output (QT): 4.0 L/min Cardiac index (QI): 2.1 L/min/m2 What is the systemic vascular resistance?

SVR = MAP - CVP / CO 86 - 6 = 80 80 / 4 = 20 mm 20 X 80 (to put into dynes) = 1600 Normal SVR = 1600

PaO2 + 30 =

SaO2

A 5' 10", 80 kg (176 lbs) male patient with multiple trauma from a motorcycle accident has developed ARDS and is being ventilated at the following settings: Arterial blood gas results reveal: pH: 7.35 PaO2: 72 torr PaCO2: 44 torr SpO2: 93% The patient is conscious and pulling on the IV lines and ventilator tubing. At this time, the respiratory therapist should recommend A. changing to assist/control mode. B. restraining the patient. C. increasing the set rate to 16 br/min. D. sedating the patient.

Sedating the patient

What do we need to give patient during IRV vent mode? (2 things)

Sedative = midazlam Paralytic = pancuronium

A-a Gradient values of MORE than 300 indicates :

Shunting

Objective information - things you can see or measure : - color - pulse - edema - BP

Signs

ECG: Has a positive P-Wave, but the spaces between each R wave is different (irregular rate)

Sinus arrhythmia

What position do we put patient in for a thoracentesis?

Sitting up and leaning forward

Adult laryngoscope blade size :

Size 3

Patient has apena during sleep for periods of 10 seconds or longer :

Sleep apnea

Polysomnography is used with :

Sleep apnea study

Forced Exp. Flow 25-75 represents what part of the airways?

Small airway

The respiratory therapist has just assisted the pulmonologist with a bedside fiberoptic bronchoscopy procedure in the ICU. In order to clean and disinfect the bronchoscope, the therapist should : A. rinse with sterile water / steam autoclave for 15 mins. B. wipe with Betadine and pasteurize for 30 minutes. C. sterilize with ethylene oxide. D. soak in alkaline glutaraldehyde for 10 hours.

Soak in alkaline glutaraldehyde for 10 hours

A 75-year-old patient with COPD is receiving oxygen at home by nasal cannula with a bubble humidifier. How should the respiratory therapist instruct the patient to clean his humidifier? A. Place it on the top shelf of the dishwasher. B. Soak it in an acetic acid solution for 20 minutes and rinse with water. C. Rinse it with distilled water and allow to air dry. D. Soak it in an alkaline glutaraldehyde solution for 30 minutes and rinse with water

Soak it in an acetic acid solution for 20 minutes and rinse with water.

Breath that patient controls all variables; ventilator does nothing :

Spontaneous breath

A patient involved in a motor vehicle accident has sustained a long bone fracture and remains in traction. The patient suddenly complains of chest pain, and becomes tachypneic and tachycardiac. After administering 100% oxygen, the therapist should recommend

Start heparin

Complete destruction of ALL microorganisms

Sterilization

Hyperlucency is seen in the soft tissue on CXR :

Subcutaeous Emphysema

Medication used for difficult intubation :

Succinylcholine

A patient is receiving mechanical ventilation in the SIMV mode. A respiratory therapist is completing a ventilator check and auscultates bilateral coarse crackles. The total rate displayed by the ventilator is 24, which is up from 14 at the last check. The patient's SpO2 is the same as it was on the last check. Which of the following should the therapist do?

Suction

Treatment for coarse crackles :

Suction

Function of the 3rd bottle in chest drainage system =

Suction control

If patient aspirates while in a position; what is the first step?

Suctioning

Treatment for L/S ratio less than 2:1

Surfactant replacement (a ratio of 2:1 or higher is good)

An adult patient is receiving PC, SIMV and the following data are observed: FIO2 = 0.80 Mandatory rate = 15 Spontaneous rate = 32 Set inspiratory pressure = 20 cm H2O PEEP = 10 cm H2O SpO2 = 92% On inspection, the patient demonstrates suprasternal retractions during spontaneous breaths. Which of the following should a respiratory therapist do NEXT? A. Decrease rise time setting. B. Switch to A/C mode. C. Increase the FIO2 to 0.90 D. Change inspiratory pressure setting to 25 cm H2O.

Switch to A/C mode (The suprasternal retractions are indicative of increased work of breathing and should be resolved by changing the mode to A/C)

A patient with COPD is receiving PC ventilation with flow triggering and has significant air trapping displayed on ventilator graphics. The patient's spontaneous breathing efforts are not always detected by the ventilator. Which of the following changes should a respiratory therapist recommend to improve patient-ventilator synchrony?

Switch to pressure trigger

Subjective information - the patient must tell you : - pain - dyspnea - nausea - muscle weakness

Symptoms

What does SIMV stand for?

Synch. Intermittent Mandatory Ventilation

Formula for pulse pressure

Systolic - Diastolic

Cavity formation on CXR :

TB

Cavity formation on xray :

TB

FRC is equal to ___ - ___ :

TLC - IC

More than 20 breaths per minute =

Tachypnea

Vibrations that are felt by the hand on the chest wall; associated with secretions.

Tactile fremitus

A patient will be away from their room for two hours while undergoing a special procedure in the radiology department. They will be using a full E cylinder. What is the maximum flow that the therapist could use without running out of oxygen?

Tank is full mean PSI = 2200 2200 X 0.3 = 660 L 600 / 120 minutes = 5.5 L / minute

A respiratory therapist is using an oxygen-calibrated flowmeter to administer a gas mixture of 70% helium and 30% oxygen. To deliver a flow of 16 L/min of the gas mixture to the patient, the therapist should set the oxygen flowmeter to :

The correction factor for a 70%/30% helium/oxygen mixture is the oxygen flow multiplied by 1.6. 10 L/min X 1.6 = 16 L/min.

Edema grading :

The higher the number, the greater the swelling

Transducer should be located at :

The level of the catheter

If you occlude the bubble humidifier and there is no "whistling sound" =

There is a leak

A pH of 7.38, PaCO2 of 46 torr, PaO2 of 41 torr are obtained on a patient who appears to be healthy. There is no tachycardia, tachypnea, or cyanosis. Which of the following is an appropriate conclusion to draw on the basis of this information? A. The blood gas sample has a gas bubble B. The blood gas sample was not mixed or warmed prior to analysis C. This is a venous blood sample D. This patient has chronic lung disease

This is a venous blood sample Bubble = CO2 down towards 0 Chronic lunger = would show s/s

Treatment for pleural effusion :

Thoracentesis Chest tube

Tiny boxes on ECG = Small boxes =

Tiny boxes on ECG = 0.04 seconds Small boxes = 0.2 seconds

What total flow is delivered to a patient if the nebulizer is set to 40% oxygen and the flowmeter is set at 9 L/min?

Total flow = FiO X flow FiO of 40% = factor of 4 4 X 9 = 36

A patient requires a total flow of 45 L / minute. What flowrate should the therapist select if the dilution control is set at 40%?

Total flow = flowmeter setting X factor Plug in the given information : 45 = flowmeter setting X 4 (factor for 40%) 45 /4 = 11.25 Answer is 12 L / minute

During inspiration valve opens and air passes through the vocal cords. During exhalation, valve closes and air passes around the DEFLATED cuff :

Trach speaking valve

Long term ventilation means we should :

Tracheostomy

Clear fluid / light straw

Transudate (seen with CHF)

Quality control test on ABG machine shows progressively increasing or decreasing but still 2 SD :

Trend DO NOTHING

You are removing the flow meter from the wall, and a massive leak occurs. What would you do?

Try reinserting the flowmeter back into the wall.

On HFOV how do we get rid of CO2 with frequency?

Turn down

Breath sound normally heard over air filled lungs indicating increased volume :

Tympanic

Aerosol device that has the highest output of all nebulizers without heat

Ultrasonic nebulizer

When would we use cardioversion (3)

Unstable A-fib Unstable A-flutter V-Tach with a pulse

If hand does not pink up within 1 -2 seconds after releasing the ulnar artery :

Use another site

A patient involved in a motor vehicle accident has sustained a long bone fracture and remains in traction. The patient suddenly complains of chest pain, and becomes tachypneic and tachycardiac. To determine the cause of the problem the respiratory therapist should recommend A. administering 100% oxygen. B. a V/Q scan. C. streptokinase. D. a STAT chest x-ray.

V / Q scan

The most common rhythm in a witnessed sudden cardiac arrest is :

V. Fib

A-a Gradient values of 66 - 300 indicates :

V/Q mismatch

Patient was on a long plane ride. Complaining of SOB, and chest pain. What would the treatment be ? (4)

V/Q scan 100% O2 Heperin Thrombolytics

How would you calculate TLC (3 ways) :

VC + RV IC + FRC IRV + FRC + Vt

VC less than ____ or ____ = vent time!

VC less than 1 L or 1000 mL = vent time!

Right before removing chest tube, the patient takes a deep breath and performs

Valsalva Manuver

In order to assist a patient with his efforts to quit smoking, the respiratory therapist could recommend : A. Varenicline B. Naloxone C. Bupropion HCI D. Albuterol

Varenicline - Chantix (cravings) Bupropion - Welbutrin (anti-depressant)

IPAP controls

Ventilation

Short acting Albuterol for severe respiratory distress :

Ventolin

Which of the following types of nebulizer can be used to deliver aerosolized medications to a ventilator circuit without altering delivered tidal volume?

Vibrating mesh

A 16 year-old patient with cystic fibrosis attends public high school. Which of the following bronchial hygiene therapies would be most appropriate for this patient? A. intrapulmonary percussive ventilation B. dornase alpha therapy C. vibratory / oscillatory PEP D. postural drainage and manual percussion

Vibratory / oscillatory PEP

What is acetic acid?

Vinegar

Decreased WBC (below 5,000) =

Viral infection

What is the amount of air that can be maximally exhaled from maximum inspiration?

Vital capacity

Dynamic compliance equation

Vt / Peak - PEEP

Picture of ventilator graphics looks like a bite was taken out of it (scalloped)

Water in the tubing = condensation = drain

Insensible water loss is :

Water loss we CANNOT measure; respirations (lungs) and perspiration (skin).

Sensible water loss is :

Water loss we can measure; urine and vomiting

Function of the 2nd bottle in chest drainage system =

Water-seal

What is a fenestrated trach tube used for?

Weaning

What is "radiodense" or "opacity" on a CXR :

White - Bone - Organs

What do patient clean home equipment with?

White distilled vinegar for 20 minutes; rinse; dry over night

Can deliver 100% body humidity?

Wick humidifier

After completing oxygen rounds, the respiratory therapist must clean flowmeters that were removed from rooms where no patients were receiving oxygen therapy. Which of the following methods is most appropriate for disinfection of the flowmeters?

Wipe with alcohol

A respiratory therapist is performing spirometry on a patient with chronic bronchitis. Which of the following equipment would produce an unreliable measurement? A. body plethysmograph B. water seal spirometer C. wright respirometer D. pressure differential pneumotachometer

Wright respirometer Least reliable

Adventitious breath sounds defines :

abnormal

PaCo2 above ______ mean respiratory failure

above 50

IPPB fails to cycle into inspiration (2) =

adjust sensitivity or tighten seal around mouth piece

A 26-year-old patient with shortness of breath is admitted to the emergency room. The patient states that he was running in Central Park with a friend and could not catch his breath. Bedside assessment reveals the following data: Pulse: 120 Respirations: 25 br/min Color: pale SpO2: 89% on room air Breath sounds: slightly diminished on the right The respiratory therapist should: A. request a STAT chest x-ray. B. administer 100% oxygen. C. insert a large bore needle into the 2nd intercostal space on the right side in the midclavicular line. D. insert a chest tube into the 2nd intercostal space on the right side in the midclavicular line

administer 100% oxygen. Take care of life function first

Hyperlucency with the lung field means (3)

air = pneumothorax, COPD, or asthma attack

While examining the chest drainage system of a mechanically-ventilated patient following thoracotomy, the respiratory therapist observes bubbling in the water-seal chamber during inspiration. This would indicate

air leaving the pleural space

To provide precise O2 concentration while using a non-rebreather - you should utilize a :

air oxygen proportioner

A 48-year-old female patient is intubated and being mechanically ventilated with a volume-cycled ventilator. The following parameters were obtained from the ventilator flow sheet: 2:00pm = Peak - 30; Plat - 25; PEEP - 5 3:00pm = Peak - 42; Plat - 30; PEEP - 10 Based on this information, the respiratory therapist should conclude that A. airway resistance is increasing. B. compliance is decreasing. C. SpO2 measurement is inconsistent with the data obtained. D. a decrease in shunt is occurring.

airway resistance is increasing 30 - 25 = 5 42 - 30 = 12

Which of the following values are needed to determine a patient's physiologic dead space?

arterial PCO2 and mixed expired PCO2

A male infant born at 36 weeks gestation was delivered by Cesarean section. The newborn is exhibiting signs of respiratory distress including nasal flaring and mild retractions. Physical examination reveals HR 160/min, RR 52/min, BP 60/40 mm Hg. The chest radiograph indicates some scattered densities. The respiratory therapist should recommend A. nasal CPAP at 5 cm H2O. B. BiPAP ventilation. C. 40% oxygen via hood. D. arterial blood gases.

arterial blood gases

Which of the following is the best index of oxygen transport for a patient who has been resuscitated after carbon monoxide poisoning? A. arterial oxygen tension B. alveolar-arterial oxygen difference C. hematocrit D. arterial oxygen content

arterial oxygen content

Overexposed Xray :

black; no blood vessels

PETCO2 is measured by

capnography

A 6-year-old child involved in a swimming pool accident is in cardiac arrest. The patient has been intubated with a size 5.5 mm endotracheal tube and has an end-tidal CO2 monitor connected to the airway. During cardiac compression the respiratory therapist notes that the end-tidal CO2 is increasing. This would indicate that A. cardiac output is increasing. B. compression depth is too low. C. endotracheal tube is too large. D. ventilation should be increased

cardiac output is increasing increase Co2 = increase CO = this is good Ventilation should increase = we don't have enough information like and ABG

A 80 kg (176 lb) male suffering from acute respiratory distress syndrome is being mechanically ventilated at the following settings: Mode: VC, A/C Tidal Volume: 500 mL Set Rate: 10 br/min Total Rate: 10 br/min PIP: 68 cm H2O FIO2: 0.45 PEEP: 8 cm H2O The following patient information is available: pH: 7.37 PaCO2: 44 torr PaO2: 89 torr HCO3-: 22 mEq/L The respiratory therapist should A. decrease PEEP. B. change to SIMV. C. add pressure support. D. change to pressure control.

change to pressure control PIP = 68 (higher than 50) Set rate and total rate are the same = patient is not breathing on their own

A 24-year-old woman with multiple trauma from a motor vehicle accident requires mechanical ventilation in the ICU. Because of a tension pneumothorax, a chest tube was inserted in the right second intercostal space and attached to a three-bottle water-seal drainage system with suction. While performing an assessment of the patient-ventilator system, the respiratory therapist notes continuous bubbling in the water seal bottle. The therapist should A. increase the suction pressure. B. clamp the chest tube. C. reduce the tidal volume. D. add water to the suction control bottle.

clamp the chest tube. Continuous bubbles in water seal = leak If you clamp the tube, you can determine if the leak is from the patient or the tube.

Aerosol device is producing short puffs means =

condensation in the tubing

A decrease in mechanical deadspeace will cause :

decrease in PaCo2

A 32-year-old victim of a motor vehicle accident (weight 80 kg, 176 lb) is on mechanical ventilation at the following settings: Tidal volume 700 mL, SIMV mandatory rate 10/min, and FIO2 0.65. As the PEEP level was increased from 17 cm H2O to 20 cm H2O, the respiratory therapist observed that the heart rate changed from 128/min to 106/min. At this time, the therapist should recommend : A. inserting a balloon tipped flow directed catheter. B. drawing an arterial blood gas. C. reassessing the patient in 20 minutes. D. decreasing the PEEP

drawing an arterial blood gas Get more info

A 32-year-old victim of a motor vehicle accident (weight 80 kg, 176 lb) is on mechanical ventilation at the following settings: Tidal volume 700 mL, SIMV mandatory rate 10/min, and FIO2 0.65. As the PEEP level was increased from 17 cm H2O to 20 cm H2O, the respiratory therapist observed that the heart rate changed from 128/min to 106/min. At this time, the therapist should recommend A. inserting a balloon tipped flow directed catheter. B. drawing an arterial blood gas. C. reassessing the patient in 20 minutes. D. decreasing the PEEP.

drawing an arterial blood gas. We need to find out why the HR has changed

Pulmonary HTN rebound is caused by :

early termination of NiO

A patient has been intubated for 4 days. When suctioning, a respiratory therapist notices that the secretions are becoming tenacious. The therapist should A. instill 5 mL normal saline into endotracheal tube. B. ensure proximal airway temperature is at least 35° C. C. switch to a heat moisture exchanger D. change device from an aerosol to a passover humidifier.

ensure airway temperature is at least 35°C

Which of the following should a respiratory therapist recommend for a patient who is receiving mechanical ventilator support and has moderate musculoskeletal pain? A. propofol (Diprivan) B. midazolam HCl (Versed) C. lorazepam (Ativan) D. fentanyl citrate (Sublimaze)

fentanyl citrate (Sublimaze)

A respiratory therapist is reviewing a chest radiograph of a patient with a hemothorax and notes the presence of the end of a chest tube. Where should the therapist expect to find the tube as it enters the chest wall?

fifth intercostal space / mid-axillary

After chest tube is removed what do we put on the opening?

gauze with vaseline

Most important pulmonary life function?

getting oxygen from the air to all of the cells in the body

A patient involved in an automobile accident is brought to the ED with tachypnea, tracheal deviation to the right, splinting, asymmetrical chest movement, and decreased breath sounds on the left side. The respiratory therapist should initially

give 100% O2

A COHb of greater than ______ % is indicative of continued smoking or environmental exposure

greater than 3%

Most sensitive sign of oxygenation

heart rate

Things that control circulation (3)

heart rate, heart strength, and cardiac output

While performing routine ventilator parameter checks on a patient on a microprocessor ventilator with a wick humidification system, the respiratory therapist notices there is very little condensation in the tubing. The most likely explanation is that the A. temperature probe is placed distal to the wye adapter. B. room temperature is lower than normal. C. heating element is not functioning properly. D. water level is just slightly above the refill line.

heating element is not functioning properly

A patient is found comatose in a car with the engine running in an enclosed garage. On arrival to the ED, the patient is receiving oxygen at 15 L/min by nonrebreathing mask and has the following vital signs: HR 147 RR 36 SpO2 100% Which tests should a respiratory therapist recommend?

hemoximetry and arterial blood gas analysis

A chest radiograph for a newly admitted patient shows honeycombing. While assessing the patient, a respiratory therapist notes a dry, non-productive cough and mild tachypnea. Which of the following diagnostic tests should the therapist recommend? A. high-resolution CT scan B. V̇/Q̇ scan C. thoracentesis D. bronchoalveolar lavage

high-resolution CT scan

Pulse ox will read ______ if carbon monoxide poisoning is present :

higher

What do you clean the trach. stoma with?

hydrogen peroxide / water mixture

All of the following would be appropriate goals for a pulmonary rehabilitation program for a patient with COPD EXCEPT A. increased exercise tolerance. B. decreased hypoxic symptoms. C. improved resting blood gases. D. improved activities of daily living.

improved resting blood gases.

A 24-year-old patient with muscular dystrophy is seen in the ED following a 2-day history of increasing shortness of breath. Blood gas analysis results obtained while the patient is breathing air are: pH 7.31 PaCO2 65 torr PaO2 58 torr HCO3- 33 mEq/L BE +4 mEq/L A respiratory therapist should recommend A. initiating invasive mechanical ventilation. B. measuring vital capacity. C. performing airway clearance. D. initiating non-invasive ventilation.

initiating non-invasive ventilation. (Non-invasive ventilation is a reasonable first step before a more aggressive step of invasive ventilation)

A 32-year-old patient with shortness of breath is admitted to the emergency room. The patient states that she was running with a friend and suddenly felt pain on the right side of her chest. Bedside assessment reveals the following data: Heart rate: 148 Respirations: 44 br/min Color: cyanotic SpO2: 70% on room air Breath sounds: absent on the right Palpation: trachea deviated to the left Percussion: tympany on the right side The respiratory therapist should first: A. request a STAT chest x-ray. B. administer IPPB stat. C. insert a 14 gauge needle into the 2nd intercostal space on the right side in the midclavicular line. D. insert a chest tube into the 2nd intercostal space on the right side in the midclavicular line.

insert a 14 gauge needle into the 2nd intercostal space on the right side in the midclavicular line. The patient is having symptoms and we know she has a pnuemo.

A 26-year-old patient with shortness of breath is admitted to the emergency room. The patient states that he was running in Central Park with a friend and could not catch his breath. Bedside assessment reveals the following data: Pulse: 45 Respirations: 50 br/min Color: cyanotic SpO2: 68% on room air Breath sounds: absent on the right Based upon the above information the respiratory therapist should recommend: A. request a STAT chest x-ray. B. administer 30% oxygen. C. administer IPPB stat. D. insert a chest tube into the 2nd intercostal space on the right side in the midclavicular line

insert a chest tube into the 2nd intercostal space on the right side in the midclavicular line The person is seriously suffering

Placement of miller blade :

insert into right side of mouth. Insert tip of blade under epoglottis

During an incentive spirometry treatment using a volume-oriented device, the patient inhales to TLC and exhales slowly to FRC. What other instructions would you give to improve distribution? A. continue with the current instructions B. switch to a flow orientated device C. initiate IPPB D. inspiratory hold

inspiratory hold Improve distribution

During mechanical ventilation, mean airway pressure will always change with a change in A. dead space. B. patient's body position. C. inspiratory time. D. FIO2

inspiratory time

RSBI for weaning =

less than 100

Normal PVR :

less than 2.5 mm

Vd / Vt for weaning =

less than 60%

A 44 week gestational age infant has just been delivered via C-section and is gasping, grunting, and has tachycardia and tachypnea. At one minute his Apgar score is 4 and at 5 minutes the score is 5. The infant is most likely suffering from

meconium aspiration

Most common hazard of suctioning :

mild bleeding (Most common - no most severe)

A patient who has ARDS is receiving PC, A/C ventilation. Radial arterial and pulmonary artery catheters are placed for monitoring. Which of the following should a respiratory therapist use to evaluate oxygen delivery in the determination of optimal PEEP for this patient? A. arterial blood gas B. mixed venous blood gas C. serum lactate level D. cardiac output

mixed venous blood gas

Establishing an airway with a suspected neck fracture :

modified jaw thrust

Best oxygen device for COPD patient who is STABLE?

nasal cannula

S1 / S2 heart sound means :

normal heart sounds If you hear S3 or S4; that's not normal - recommend Echo.

Double spike on PA catheter :

normal; this is the diacritic dotch

The repeated administration of fluticasone propionate (Flovent) by inhalation is associated with which of the following adverse effects? A. cardiac dysrhythmias B. rebound congestion C. oral candidiasis D. dry mouth

oral candidiasis

Which of the following should be used for high-level disinfection of nondisposable ventilator tubing? A. pasteurization B. isopropyl alcohol C. ionizing radiation D. hydrogen peroxide

pasteurization

While performing a patient-ventilator check, a respiratory therapist observes very little condensation in the heated wire circuit. The heated wick humidifier contains an appropriate amount of water. The most likely explanation is that the : A. minute ventilation is greater than 15 L/min. B. patient circuit is operating normally. C. flow is set at too low of a value. D. room temperature is lower than normal.

patient circuit is operating normally.

#1 indication for stress test :

patient says i have trouble breathing on exertion

An adult patient is intubated after being pulseless for several minutes. An exhaled CO2 detection device fails to change color despite confirmation of tracheal placement by auscultation and chest rise. A respiratory therapist should recommend : A. performing a direct laryngoscopy. B. replacing the CO2 detection device. C. obtaining a stat chest radiograph. D. performing pulse oximetry.

performing a direct laryngoscopy (Direct visualization of the larynx with a laryngoscope will confirm the tube has passed through the cords)

A patient is receiving PC ventilation and the following data are noted: PIP = 52 cm H2O Exhaled VT = 300 mL HR = 175 SpO2 = 79 The patient has become agitated and tympany is noted on the left. A respiratory therapist should recommend : A. initiating VC ventilation. B. administering nitric oxide. C. obtaining an arterial blood gas analysis. D. performing needle decompression.

performing needle decompression (The onset of agitation, tachycardia, oxygen desaturation, and left chest tympany are suggestive of a tension pneumothorax)

A respiratory therapist is assisting a physician with a tracheostomy for a patient who is receiving PC ventilation. Following percutaneous placement of a tracheostomy tube, the therapist observes increasing heart rate, decreasing exhaled tidal volume, and increasingly distant breath sounds over the right chest. The therapist should anticipate treatment for :

peumothorax

The major component of pulmonary surfactant is : A. protein. B. glucose. C. phospholipid. D. polysaccharide.

phospholipid

Costophrenic angles are obliterated (blunted) on CXR :

pleural effusion

If the vent fails to cycle - what do we check?

power supply

AVR is the only limb lead of an ECG that :

produces an upside down (negative) pattern

If a patient is on a vent and you need to fix an alarm (troubleshoot) - we always do what FIRST

provide manual ventilation first

Green sputum =

pseudomonas

Pink sputum =

pulmonary edema

If spontaneously breathing patient on chest tube becomes disconnected - what do we do?

put chest tube in glass of water

A 44-year-old patient presents to the ED with diabetic ketoacidosis. Which of the following respiratory patterns is most consistent with this diagnosis? A. irregular breathing with periods of apnea B. regular, slow, deep breathing C. rapid, deep breathing D. slow, shallow breathing

rapid, deep breathing (An increase in rate and depth of breathing, called Kussmaul's breathing)

A 2-month-old infant is receiving 40% oxygen via an oxygen hood. While performing oxygen rounds, the respiratory therapist analyzes the FIO2 inside the hood and discovers that the FIO2 is registering 0.50. The respiratory therapist calibrated the oxygen analyzer at the beginning of the shift. The respiratory therapist should A. check to see that the hood is sealed tightly around the infant's neck. B. decrease the flow to the hood. C. check to see that the temperature setting on the humidifier is at 34oC. D. recalibrate the oxygen analyzer.

recalibrate the oxygen analyzer.

Immediately after extubation of a patient in the ICU, the respiratory therapist observes increasing respiratory distress with intercostal retractions and marked stridor. The SpO2 on 40% oxygen is noted to be 86%. Which of the following would be most appropriate at this time? A. cool mist aerosol treatment B. aerosolized racemic epinephrine C. manual ventilation with resuscitation bag and mask D. reintubation

reintubation

What do we need to do before using trach button :

remove trach tube

A post-operative thoracotomy patient is receiving mechanical ventilation in the recovery room with a tidal volume of 750 mL, SIMV mandatory rate 8/min, FIO2 of 0.40. Arterial blood gas results show: pH: 7.36 PaO2: 89 torr PaCO2: 45 torr SaO2: 95% The patient is breathing fast and shallow. Based upon this information, the respiratory therapist should recommend A. repeating the arterial blood gas in 30 minutes. B. increase SIMV mandatory rate to 10/min. C. sedating the patient. D. paralyzing the patient with pancuronium bromide

repeating the arterial blood gas in 30 minutes we are trying to wean this patient; give them some time.

A 26-year-old patient with shortness of breath is admitted to the emergency room. The patient states that he was running in Central Park with a friend and could not catch his breath. Bedside assessment reveals the following data: Pulse: 134 Respirations: 35 Color: pale SpO2: 91% on room air Breath sounds: diminished on the right The respiratory therapist should: A. request a STAT chest x-ray. B. recommend a V/Q scan C. insert a large bore needle into the 2nd intercostal space on the right side in the midclavicular line. D. insert a chest tube into the 2nd intercostal space on the right side in the midclavicular line

request a STAT chest x-ray "soft signs" - need more info

After soaking item in Alkaline Glutaradehyde - we must?

rinse with sterile water

APGAR score : Grimace

score = 1

APGAR score : Pulse is less than 100 / min.

score = 1

APGAR score : Some flexion of extremities

score = 1

APGAR score : Cough or sneeze

score = 2

Breath sounds : Crackles (rales) - indicates

secretions / fluids

A 25-year-old patient with apnea is receiving PC ventilation. Arterial blood gas results are as follows: pH = 7.20 PCO2 = 65 torr PO2 = 70 torr HCO3- = 25 mEq/L BE = -4 mEq/L A respiratory therapist should recommend increasing the : A. set inspiratory pressure. B. expiratory time. C. sensitivity. D. peak flow.

set inspiratory pressure

Lethargic, somnolent, sleepy - consider ________ or ________ in patients with COPD

sleep apena excessive O2 therapy in COPD patients

When you increase IPAP; always pick the answer with ________ :

start with lower increase

A 56-year-old male patient who weighs 90 kg (198 lb) is receiving mechanical ventilation in the PC, SIMV mode and the following data is available: FIO2: 1.00 Set rate: 20 /min. Total rate: 20 /min. PEEP: 18 cm H2O Peak pressure: 35 cm H2O Arterial blood gas results show: pH: 7.35 PaCO2: 45 torr PaO2: 43 torr HCO3-: 22 mEq/L It would be appropriate for the respiratory therapist to A. increase the set rate to 25 /min. B. switch to inverse ratio ventilation. C. increase pressure to 40 cm H2O. D. increase PEEP to 25 cm H2O. `

switch to inverse ratio ventilation. FIO2 is already 100% PEEP is already high this patient clearly has ARDS = inverse ratio

Which of the following will determine the aortic pulse pressure? A. systolic + diastolic + diastolic / 3 B. diastolic + pulse pressure / 3 C. systolic pressure - diastolic pressure D. stroke volume x heart rate x 10

systolic pressure - diastolic pressure

The transducer should be at _____________

the same level as the tip of the catheter

After injecting a small amount of air into the balloon of a pulmonary artery catheter, the respiratory therapist sees a small amplitude change with the mean pressure reading 2 points below the PA end-diastolic pressure. Based upon this information, the therapist should conclude that A. there is pressure dampening. B. the transducer is placed too high. C. there is an obstruction in the catheter. D. this is a normal wedge tracing.

this is a normal wedge tracing.

Confirm Dx of PDA in baby?

umbilical artery sample

What mode of ventilation would we use for flail chest?

volume control

Underexposed Xray :

white; cannot see the vertebra discs through the heart shadow

An HME is most appropriate for a patient : A. who is receiving NPPV. B. with ARDS who is receiving mechanical ventilation. C. who is receiving mechanical ventilation in the PACU. D. with a bronchopleural fistula who is receiving mechanical ventilation.

who is receiving mechanical ventilation in the PACU (An HME should be considered for short-term mechanical ventilation)

Post-operative IS / SMI goal =

½ of the pre-operative inspiratory capacity


Conjuntos de estudio relacionados

Astronomy: Solar System Vocabulary

View Set

38 Med Surg 8 Assessment of the Gastrointestinal System,

View Set

CH 1: Introduction to Child Heath and Pediatric Nursing

View Set

ECON 300 EXAM 3 REVIEW QUESTIONS

View Set